June 2007 - Sec 1 - Game 1 - Q5

Video Transcript:

0:08
Then turning our attention to the fifth and final question of the first game, "Which of
0:12
the following must be true about any acceptable product code . . . " We're looking again for
0:17
what must be true.
0:19
(A), There is exactly 1 digit between the 0 and the 1. Again, looking at our free
0:25
hypothetical that we just drew up from Question 4, you notice that there are no digits between
0:33
the 0 and the 1, so (A) does not have to be true, and (A) would be eliminated.
0:38
(B), There is exactly one digit between the 1 and the 2. Again, that does not have
0:44
to be true, looking at our scenario 1, and our hypothetical scenario 1 from Question
0:51
3, we notice that there is no space between 1 and 2. No digits, they are directly next
0:57
to each other, so (B) does not follow, and (B) would be eliminated.
1:03
(C), There are at most two digits between the 1 and the 3. Again, you notice from
1:11
scenario 1, we could actually have three digits in between the 1 and the 3. Have 1 be first,
1:20
3 be fifth, and there would be one, two, three digits in between, so (C) does not have to
1:29
be true, and (C) would be eliminated.
1:31
Moving to (D), There are at most, two digits between the 2 and the 3. Again, looking
1:38
at scenario 2, we see we could have three digits between the 2 and the 3. 2 could be
1:44
first, 3 could be fifth. Again, (D) does not have to be true, so (D) is out.
1:51
Which brings this process of elimination to (E), ?There are at most two digits between
1:56
the 2 and the 4. If you notice in scenario 1 where 2 is second, the latest we could have
2:02
4 is fifth, and that is exactly two spaces in between. At most, two, in that scenario.
2:12
Whereas in scenario 1, there is no space between 2 and 4. You notice there is at most two digits
2:19
between 2 and 4. So (E) must be true, and (E) would be the correct answer.
2:25
Alright, I hope that was helpful. Let's turn our attention to Game 2.